Đến nội dung

phongmaths

phongmaths

Đăng ký: 25-09-2017
Offline Đăng nhập: 07-08-2019 - 22:27
-----

#723767 Cho x+y=2. Chứng minh x^5 + y^5 ≥ 2

Gửi bởi phongmaths trong 14-07-2019 - 21:30

Ta có 

$2(a^5+b^5)=(a^5+b^5)(a+b)\geq (a^3+b^3)^2$

$(a^5+b^5)\geq \frac{(a^3+b^3)^2}{2}$

Mà $2(a^3+b^3)=(a+b)(a^3+b^3)\geq (a^2+b^2)^2$

$\Rightarrow a^3+b^3\geq \frac{(a^2+b^2)^2}{2}$

$\Rightarrow a^5+b^5\geq\frac{( \frac{(a^2+b^2)^2}{2})^2}{2}=\frac{(a^2+b^2)^4}{8}\geq \frac{(\frac{(a+b)^2}{2})^4}{8} =\frac{(\frac{(2)^2}{2})^4}{8}=2$

Dấu bằng xảy ra khi a=b=1




#723762 Bất đẳng thức

Gửi bởi phongmaths trong 14-07-2019 - 15:02

Ta có Do $a^{2}+b^{2}+c^{2}=1$ nên 

$\frac{bc}{a^{2}+1}=\frac{bc}{a^{2}+a^2+b^2+c^2}=\frac{bc}{(a^2+b^2)+(a^2+c^2)}\leq \frac{1}{4}(\frac{(b+c)^2}{(a^2+b^2)+(a^2+c^2)})$

Áp dụng BĐT Cauchy-Schwart ta có

 $\frac{bc}{a^{2}+1}\leq \frac{1}{4}(\frac{(b+c)^2}{(a^2+b^2)+(a^2+c^2)})\leq \frac{1}{4}(\frac{b^2}{a^2+b^2}+\frac{c^2}{a^2+c^2})$

CMTT suy ra $\frac{ac}{b^{2}+1}\leq \frac{1}{4}(\frac{a^2}{b^2+a^2}+\frac{c^2}{b^2+c^2}),\frac{ab}{c^{2}+1}\leq \frac{1}{4}(\frac{a^2}{a^2+c^2}+\frac{b^2}{b^2+c^2})$

$\frac{bc}{a^{2}+1}+\frac{ac}{b^{2}+1}+\frac{ab}{c^{2}+1}\leq \frac{1}{4}(\sum (\frac{b^2}{a^2+b^2}+\frac{a^2}{b^2+a^2}))=\frac{3}{4}$

Dấu bằng xảy ra khi $a=b=c=\frac{1}{\sqrt{3}}$




#723760 4($\sum \frac{1}{a+b})-\sum \frac{1}{a}\leqslan...

Gửi bởi phongmaths trong 14-07-2019 - 10:44

Ta có $(a+b+c)^4=a^4+b^4+c^4+12abc(a+b+c)+4ab(a^2+b^2)+4bc(b^2+c^2)+4ca(c^2+a^2)+6(a^2b^2+b^2c^2+c^2a^2)$

BĐT phải chứng minh là 

$8(a^{4}+b^{4}+c^{4})+19abc(a+b+c)\geq a^4+b^4+c^4+12abc(a+b+c)+4ab(a^2+b^2)+4bc(b^2+c^2)+4ca(c^2+a^2)+6(a^2b^2+b^2c^2+c^2a^2)$

$\Leftrightarrow 7(a^4+b^4+c^4+abc(a+b+c))\geq 4ab(a^2+b^2)+4bc(b^2+c^2)+4ca(c^2+a^2)+6(a^2b^2+b^2c^2+c^2a^2)$

Áp dụng BĐT Schur bậc 4 ta có $a^4+b^4+c^4+abc(a+b+c)\geq ab(a^2+b^2)+bc(b^2+c^2)+ca(c^2+a^2)$

$\Rightarrow VT\geq 7(ab(a^2+b^2)+bc(b^2+c^2)+ca(c^2+a^2))=4(ab(a^2+b^2)+bc(b^2+c^2)+ca(c^2+a^2))+3(ab(a^2+b^2)+bc(b^2+c^2)+ca(c^2+a^2))\geq 4(ab(a^2+b^2)+bc(b^2+c^2)+ca(c^2+a^2))+6(a^2b^2+b^2c^2+c^2a^2)=VP$

Dấu bằng xảy ra khi a=b=c  hoặc a=b,c=0 và các hoán vị




#723704 $\sum \frac{a+b}{c} \geq 2\sqrt{( \sum a)(...

Gửi bởi phongmaths trong 12-07-2019 - 22:41

Ta có: Không mất tính tổng quát, giả sử c nằm giữa a và b

Áp dụng BĐT AM-GM ta có 

$2\sqrt{(a+b+c)(\frac{a}{bc}+\frac{b}{ca}+\frac{c}{ab})}=2\sqrt{\frac{(a+b+c)}{c}.c(\frac{a}{bc}+\frac{b}{ca}+\frac{c}{ab})}\leq \frac{(a+b+c)}{c}+c(\frac{a}{bc}+\frac{b}{ca}+\frac{c}{ab})=\frac{a+b}{c}+\frac{a}{b}+\frac{b}{a}+1+\frac{c^2}{ab}$

BĐT cần chứng minh là $\frac{a+b}{c}+\frac{a}{b}+\frac{b}{a}+1+\frac{c^2}{ab}\leq \frac{a+b}{c}+\frac{b+c}{a}+\frac{c+a}{b}\Leftrightarrow 1+\frac{c^2}{ab}\leq \frac{c}{a}+\frac{c}{b}\Rightarrow (\frac{c}{a}-1)(\frac{c}{b}-1)\leq 0 \Leftrightarrow \frac{(c-a)(c-b)}{ab}\leq 0$(BĐT đúng do c nằm giữa a và b )

Dấu bằng xảy ra khi a=b=c




#723542 Hỏi về bất đẳng thức

Gửi bởi phongmaths trong 05-07-2019 - 20:28

Ta có 

BĐT cần chứng minh

$\frac{(\sqrt{ab}+\sqrt{bc}+\sqrt{ca})^2}{2(a+b+c)}\geq \sqrt{ab}+\sqrt{bc}+\sqrt{ca} -\frac{a+b+c}{2}$

$\Leftrightarrow \frac{(\sqrt{ab}+\sqrt{bc}+\sqrt{ca})^2}{2(a+b+c)}+\frac{a+b+c}{2}\geq \sqrt{ab}+\sqrt{bc}+\sqrt{ca}$

Áp dụng BĐT AM-GM ta có 

$\frac{(\sqrt{ab}+\sqrt{bc}+\sqrt{ca})^2}{2(a+b+c)}+\frac{a+b+c}{2}\geq 2\sqrt{\frac{(\sqrt{ab}+\sqrt{bc}+\sqrt{ca})^2}{2(a+b+c)}.\frac{a+b+c}{2}} = \sqrt{ab}+\sqrt{bc}+\sqrt{ca}$

Dấu bằng xảy ra khi a=b=c




#723524 Cho các số không âm a,b,c

Gửi bởi phongmaths trong 04-07-2019 - 21:52

Cho các số không âm a, b, c không có hai số nào đồng thời bằng 0. Chứng minh bất đẳng thức sau 

          $\frac{1}{\sqrt{a^2+bc}}+\frac{1}{\sqrt{b^2+ca}}+\frac{1}{\sqrt{c^2+ab}}\geq \frac{2\sqrt{2}}{\sqrt{ab+bc+ca}}$




#723523 Cho a,b,c là các số thực dương.Chứng minh bất đẳng thức $\sum...

Gửi bởi phongmaths trong 04-07-2019 - 21:41

Ta có 

$\frac{a^{4}}{a^{2}+ab+b^{2}}-\frac{ab^3}{a^{2}+ab+b^{2}}=\frac{a(a-b)(a^2+ab+b^2)}{a^{2}+ab+b^{2}}=a^2-ab$

CMTT ta suy ra $\sum \frac{a^{4}}{a^{2}+ab+b^{2}}=\sum (\frac{a^{4}}{a^{2}+ab+b^{2}}-\frac{ab^3}{a^2+ab+b^2})+\sum \frac{ab^3}{a^2+ab+b^2}=\sum (a^2-ab)+\sum \frac{ab^3}{a^2+ab+b^2}=(a^2+b^2+c^2-ab-bc-ca)+\sum \frac{ab^3}{a^2+ab+b^2}$

Mà $a^3+b^3+c^3-3abc=(a+b+c)(a^2+b^2+c^2-ab-bc-ca)$

$\Rightarrow \frac{a^{3}+b^{3}+c^{3}}{a+b+c}=\frac{a^{3}+b^{3}+c^{3}-3abc}{a+b+c}+\frac{3abc}{a+b+c}=\frac{(a+b+c)(a^2+b^2+c^2-ab-bc-ca)}{a+b+c}+\frac{3abc}{a+b+c}=(a^2+b^2+c^2-ab-bc-ca)+\frac{3abc}{a+b+c}$

BĐT phải chứng minh là $(a^2+b^2+c^2-ab-bc-ca)+\sum \frac{ab^3}{a^2+ab+b^2}\geq (a^2+b^2+c^2-ab-bc-ca)+\frac{3abc}{a+b+c}$

$\Leftrightarrow \frac{ab^3}{a^2+ab+b^2}+\frac{bc^3}{b^2+bc+c^2}+\frac{ca^3}{c^2+ca+a^2}\geq \frac{3abc}{a+b+c}$

Chia cả hai vế cho abc ta được

$\frac{b^2}{c(a^2+ab+b^2)}+\frac{c^2}{a(b^2+bc+c^2)}+\frac{a^2}{b(c^2+ca+a^2)}\geq \frac{3}{a+b+c}$

Áp dụng BĐT Cauchy schwart ta được

$\frac{b^2}{c(a^2+ab+b^2)}+\frac{c^2}{a(b^2+bc+c^2)}+\frac{a^2}{b(c^2+ca+a^2)}\geq \frac{(a+b+c)^2}{c(a^2+ab+b^2)+a(b^2+bc+c^2)+b(c^2+ca+a^2)}=\frac{(a+b+c)^2}{(a+b+c)(ab+bc+ca)}=\frac{a+b+c}{ab+bc+ca}$

Mà $3(ab+bc+ca)\leq (a+b+c)^2$

$\Rightarrow \frac{a+b+c}{ab+bc+ca}\geq \frac{3}{a+b+c}$(đpcm)

Dấu bằng xảy ra khi a=b=c




#723216 Có bạn nào giải được câu hỏi này không ?

Gửi bởi phongmaths trong 20-06-2019 - 19:45

số 4 đó bạn. Đây chính là dãy số Fibonacci




#723190 Chứng minh NP, RH, AB đồng qui

Gửi bởi phongmaths trong 19-06-2019 - 23:38

Ta có

$\angle AMF=\angle AHF$(cùng chắn cung AF )

Mà BFHD là tứ giác nội tiếp nên $\angle AHF =\angle ABD$

Tứ giác AEBD là tứ giác nội tiếp nên $\angle ABD =\angle AEP\Rightarrow \angle AMF=\angle AEP$

nên tứ giác MNEP là tứ giác nội tiếp $\Rightarrow \angle NPE =\angle NME=\angle FME=\angle FAE=\angle EDC$

$\Rightarrow NP//BC$

Gọi  G là giao điểm của NP với AB 

$\Rightarrow \angle AGP=\angle ABC =\angle AEP$

Nên AGEP là tứ giác nội tiếp 

$\Rightarrow AN.NE=GN.NP$ (phương tích trong)

Mà ARERQ là tứ giác nội tiếp (AEF) $\Rightarrow RN.NQ=AN.AE$(phương tích trong)

$\Rightarrow RN.NQ=GN.NP$

Nên RGQP là tứ giác nội tiếp 

$\Rightarrow \angle GRQ =\angle GPQ$ (1)

Gọi I là giao điểm của PH và BC 

Do $\angle AQH=90^{\circ}$(chắn nữa đường tròn)

Nên AQDI là tứ giác nội tiếp 

$\Rightarrow \angle QAH =\angle QID$

Mà GP//BC nên $\angle GPH =\angle PID\Rightarrow \angle GPH =\angle HAQ$

Mà $\angle HAQ =\angle HRQ$ (cùng chắn cung HQ) 

$\Rightarrow \angle GPH=\angle HRQ$ (2)

Từ (1) và (2) $\Rightarrow \angle QRG=\angle QRH$

Suy ra R,G,H thẳng hàng 

Suy ra RH,GP,AB đồng quy

ps:xin lỗi vì không biết đính kèm hình  :icon6:




#723176 cho a,b,c,d là các số nguyên dương đôi một khác nhau có $\frac...

Gửi bởi phongmaths trong 19-06-2019 - 15:29

cho a,b,c,d là các số nguyên dương đôi một khác nhau có $\frac{a}{a+b}+\frac{b}{b+c}+\frac{c}{c+d}+\frac{d}{a+d}=2$ cm abcd là SCP

Ta có 

$\frac{a}{a+b}+\frac{b}{b+c}+\frac{c}{c+d}+\frac{d}{a+d}=2\Leftrightarrow \frac{b}{b+c}+\frac{d}{a+d}=1-\frac{a}{a+b}+1-\frac{c}{c+d} \Leftrightarrow \frac{b}{b+c}+\frac{d}{a+d}=\frac{b}{a+b}+\frac{d}{c+d}\Leftrightarrow b(\frac{1}{b+c}-\frac{1}{a+b})+d(\frac{1}{a+d}-\frac{1}{c+d})=0\Leftrightarrow \frac{b(a-c)}{(b+c)(a+b)}+\frac{d(c-a)}{(d+a)(c+d)}=0\Leftrightarrow (a-c)(\frac{b}{(b+c)(a+b)}-\frac{d}{(d+a)(c+d)})=0\Leftrightarrow (a-c)(\frac{b(d+a)(c+d)-d(b+c)(a+b)}{(a+b)(b+c)(c+d)(d+a)})=0\Leftrightarrow \frac{(a-c)(b-d)(ac-bd)}{(a+b)(b+c)(c+d)(d+a)}=0$

Do a,b,c,d khác nhau từng đôi một nên $a-c\neq 0,b-d\neq 0$

$\Rightarrow ac-bd=0 \Leftrightarrow ad=bc$

$\Rightarrow abcd=(ac)^2$ là số chính phương




#723172 Chứng minh rằng $(\frac{4}{a^2+b^2}+1)(\fr...

Gửi bởi phongmaths trong 19-06-2019 - 15:12

Ta có 

Áp dụng BĐT Bunhiacopxki ta có $3(a+b+c)^{2}\leq 3.3(a^2+b^2+c^2)=3.3.3=27$

BĐT cần chứng minh là $(\frac{4}{a^2+b^2}+1)(\frac{4}{b^2+c^{2}}+1)(\frac{4}{a^2+c^2}+1)\geq 27$

Đặt $a^2+b^2=x,b^2+c^2=y,c^2+a^2=z$ thì x+y+z=6 

BĐT cần chứng minh tương đương với $(\frac{4}{x}+1)(\frac{4}{y}+1)(\frac{4}{z}+1)\geq 27 \Leftrightarrow (x+4)(y+4)(z+4)\geq 27xyz$

Áp dụng BĐT AM-GM ta có $x+4=x+2+2\geq 3\sqrt[3]{2.2.x}=3\sqrt[3]{4x}$

CMTT ta được $y+4\geq 3\sqrt[3]{4y}, z+4\geq 3\sqrt[3]{4z}$

$\Rightarrow (x+4)(y+4)(z+4)\geq 27\sqrt[3]{4^3xyz}$

BĐT cần chứng minh là $\sqrt[3]{4^3xyz}\geq xyz \Leftrightarrow 2\geq \sqrt[3]{xyz}$

Do x+y+z=6 $\Rightarrow 6=x+y+z\geq 3\sqrt[3]{xyz}\Leftrightarrow 2\geq \sqrt[3]{xyz}$ 

Dấu bằng xảy ra khi x=y=z=2 khi a=b=c=1




#723170 $\left [ \sum \frac{(a+b)(a^2-ab+b^2)^2}{a...

Gửi bởi phongmaths trong 19-06-2019 - 14:58

Ta có  $a^2-ab+b^2\geq \frac{1}{3}(a^2+ab+b^2)$ 

Đặt $B=\frac{(a+b)(a^2-ab+b^2)^2}{a^2+ab+b^2}+\frac{(b+c)(b^2-bc+c^2)^2}{b^2+bc+c^2}+\frac{(c+a)(c^2-ca+a^2)^2}{c^2+ca+a^2}$

$\Rightarrow B=\sum \frac{(a+b)(a^2-ab+b^2)^2}{a^2+ab+b^2} \geq \sum \frac{\frac{1}{3}(a+b)(a^2-ab+b^2)(a^2+ab+b^2)}{a^2+ab+b^2}=\frac{1}{3}(\sum (a+b)(a^2-ab+b^2))=\frac{1}{3}(\sum a^3+b^3)=\frac{2}{3}(a^3+b^3+c^3)$

Mà do $\frac{abc}{ab+bc+ca}\geq \frac{1}{9} \Rightarrow 9abc\geq ab+bc+ca \Rightarrow ab+bc+ca \leq 9\sqrt{(\frac{ab+bc+ca}{3})^3} \Rightarrow ab+bc+ca \geq \frac{1}{3}$ 

áp dụng BĐT AM-GM ta có $a^3+b^3+c^3 \geq 3abc\geq \frac{abc}{ab+bc+ca}$

$\Rightarrow B\geq \frac{2}{3}\frac{abc}{ab+bc+ca}$

$\Rightarrow A\geq \frac{2}{3}\frac{abc}{ab+bc+ca}+\frac{2(ab+bc+ca)}{243abc}=\frac{2}{243}(\frac{81abc}{ab+bc+ca}+\frac{(ab+bc+ca)}{abc})\geq \frac{2}{243}.2\sqrt{\frac{81abc}{ab+bc+ca}.\frac{(ab+bc+ca)}{abc}}=\frac{4}{27}$

Dấu bằng xảy ra khi $a=b=c=\frac{1}{3}$




#723168 cho các số thực dương x,y,z thỏa mãn xy+yz+xz=3.Chứng minh bất đẳng thức...

Gửi bởi phongmaths trong 19-06-2019 - 14:25

Ta có 

$\sum \frac{x^2}{\sqrt{x^3+8}}= \sum \frac{x^2}{(x+2)(x^2-2x+4)}\geq \sum \frac{2x^2}{x+2+x^2-2x+4}=\sum \frac{2x^2}{x^2-x+6}$

Áp dụng BĐT cauchy-Schwart  và với xy+yz+zx=3 ta có 

$\sum \frac{x^2}{\sqrt{x^3+8}}\geq 2\frac{(x+y+z)^2}{x^2+y^2+z^2-x-y-z+18}=2\frac{(x+y+z)^2}{x^2+y^2+z^2-x-y-z+12+2(xy+yz+zx)}=2\frac{(x+y+z)^2}{(x+y+z)^2-x-y-z+12}$

Đặt t=x+y+z với $t\geq \sqrt{3(xy+yz+zx)}=3$

BĐT cần chứng minh là $\frac{2t^2}{t^2-t+12}\geq 1\Leftrightarrow (t-3)(t+4)\geq 0 $ (BĐT đúng )

Dấu bằng xảy ra khi t=3 khi x=y=z=1




#723156 cho các số thực dương x,y,z thỏa mãn xy+yz+xz=3.Chứng minh bất đẳng thức...

Gửi bởi phongmaths trong 18-06-2019 - 22:43

cho các số thực dương  x,y,z thỏa mãn xy+xz+xz=3.Chứng minh bất đẳng thức$\sum \frac{x^2}{\sqrt{x^3+8}}\geq 1.$

sao lại là xy+xz+xz=3




#722694 cho ab+bc+ca=1

Gửi bởi phongmaths trong 03-06-2019 - 19:08

We have $1+a^2=ab+bc+ac+a^2=(a+b)(a+c)$

Infer  $$P=\sum\dfrac{a}{(a+b)(a+c)}=\dfrac{2(ab+bc+ac)}{(a+b)(b+c)(c+a)}$$

Notice $$(a+b)(b+c)(c+a)\ge\dfrac{8}{9} (ab+bc+ac)(a+b+c)\ge\dfrac{8}{3} (ab+bc+ac)$$

So that  $$P\le\dfrac{3}{4}$$

đoạn đó phải là $P= \frac{a}{(a+b)(a+c)}+\frac{b}{(b+c)(b+a)}-\frac{1}{(c+a)(c+b)}$